If $a, b, c$ are in geometric progression and $a+b+c=abc$ then what is minimum value of $a^4+a^2+7$?











up vote
-2
down vote

favorite
1












If $a, b, c$ are in geometric progression and $a+b+c=abc$ then what is minimum value of $a^4+a^2+7$?(a,b,c are positive real numbers)










share|cite|improve this question









New contributor




Harry Potter is a new contributor to this site. Take care in asking for clarification, commenting, and answering.
Check out our Code of Conduct.
















  • 4




    Hint: Let $b=ar$ and $c=ar^2$.
    – Anurag A
    Nov 15 at 9:23










  • Welcome to MSE. It will be more likely that you will get an answer if you show us that you made an effort. This should be added to the question rather than in the comments.
    – José Carlos Santos
    Nov 15 at 9:25










  • Contest question? What's the source, please?
    – Gerry Myerson
    Nov 15 at 9:27










  • Are $a, b, c$ integers?
    – Mees de Vries
    Nov 15 at 9:39






  • 1




    The fact that the solution is trivial (despite your late addition of constraints) makes the problem statement suspicious. Double check it.
    – Yves Daoust
    Nov 15 at 10:05















up vote
-2
down vote

favorite
1












If $a, b, c$ are in geometric progression and $a+b+c=abc$ then what is minimum value of $a^4+a^2+7$?(a,b,c are positive real numbers)










share|cite|improve this question









New contributor




Harry Potter is a new contributor to this site. Take care in asking for clarification, commenting, and answering.
Check out our Code of Conduct.
















  • 4




    Hint: Let $b=ar$ and $c=ar^2$.
    – Anurag A
    Nov 15 at 9:23










  • Welcome to MSE. It will be more likely that you will get an answer if you show us that you made an effort. This should be added to the question rather than in the comments.
    – José Carlos Santos
    Nov 15 at 9:25










  • Contest question? What's the source, please?
    – Gerry Myerson
    Nov 15 at 9:27










  • Are $a, b, c$ integers?
    – Mees de Vries
    Nov 15 at 9:39






  • 1




    The fact that the solution is trivial (despite your late addition of constraints) makes the problem statement suspicious. Double check it.
    – Yves Daoust
    Nov 15 at 10:05













up vote
-2
down vote

favorite
1









up vote
-2
down vote

favorite
1






1





If $a, b, c$ are in geometric progression and $a+b+c=abc$ then what is minimum value of $a^4+a^2+7$?(a,b,c are positive real numbers)










share|cite|improve this question









New contributor




Harry Potter is a new contributor to this site. Take care in asking for clarification, commenting, and answering.
Check out our Code of Conduct.











If $a, b, c$ are in geometric progression and $a+b+c=abc$ then what is minimum value of $a^4+a^2+7$?(a,b,c are positive real numbers)







sequences-and-series






share|cite|improve this question









New contributor




Harry Potter is a new contributor to this site. Take care in asking for clarification, commenting, and answering.
Check out our Code of Conduct.











share|cite|improve this question









New contributor




Harry Potter is a new contributor to this site. Take care in asking for clarification, commenting, and answering.
Check out our Code of Conduct.









share|cite|improve this question




share|cite|improve this question








edited Nov 15 at 9:48





















New contributor




Harry Potter is a new contributor to this site. Take care in asking for clarification, commenting, and answering.
Check out our Code of Conduct.









asked Nov 15 at 9:21









Harry Potter

396




396




New contributor




Harry Potter is a new contributor to this site. Take care in asking for clarification, commenting, and answering.
Check out our Code of Conduct.





New contributor





Harry Potter is a new contributor to this site. Take care in asking for clarification, commenting, and answering.
Check out our Code of Conduct.






Harry Potter is a new contributor to this site. Take care in asking for clarification, commenting, and answering.
Check out our Code of Conduct.








  • 4




    Hint: Let $b=ar$ and $c=ar^2$.
    – Anurag A
    Nov 15 at 9:23










  • Welcome to MSE. It will be more likely that you will get an answer if you show us that you made an effort. This should be added to the question rather than in the comments.
    – José Carlos Santos
    Nov 15 at 9:25










  • Contest question? What's the source, please?
    – Gerry Myerson
    Nov 15 at 9:27










  • Are $a, b, c$ integers?
    – Mees de Vries
    Nov 15 at 9:39






  • 1




    The fact that the solution is trivial (despite your late addition of constraints) makes the problem statement suspicious. Double check it.
    – Yves Daoust
    Nov 15 at 10:05














  • 4




    Hint: Let $b=ar$ and $c=ar^2$.
    – Anurag A
    Nov 15 at 9:23










  • Welcome to MSE. It will be more likely that you will get an answer if you show us that you made an effort. This should be added to the question rather than in the comments.
    – José Carlos Santos
    Nov 15 at 9:25










  • Contest question? What's the source, please?
    – Gerry Myerson
    Nov 15 at 9:27










  • Are $a, b, c$ integers?
    – Mees de Vries
    Nov 15 at 9:39






  • 1




    The fact that the solution is trivial (despite your late addition of constraints) makes the problem statement suspicious. Double check it.
    – Yves Daoust
    Nov 15 at 10:05








4




4




Hint: Let $b=ar$ and $c=ar^2$.
– Anurag A
Nov 15 at 9:23




Hint: Let $b=ar$ and $c=ar^2$.
– Anurag A
Nov 15 at 9:23












Welcome to MSE. It will be more likely that you will get an answer if you show us that you made an effort. This should be added to the question rather than in the comments.
– José Carlos Santos
Nov 15 at 9:25




Welcome to MSE. It will be more likely that you will get an answer if you show us that you made an effort. This should be added to the question rather than in the comments.
– José Carlos Santos
Nov 15 at 9:25












Contest question? What's the source, please?
– Gerry Myerson
Nov 15 at 9:27




Contest question? What's the source, please?
– Gerry Myerson
Nov 15 at 9:27












Are $a, b, c$ integers?
– Mees de Vries
Nov 15 at 9:39




Are $a, b, c$ integers?
– Mees de Vries
Nov 15 at 9:39




1




1




The fact that the solution is trivial (despite your late addition of constraints) makes the problem statement suspicious. Double check it.
– Yves Daoust
Nov 15 at 10:05




The fact that the solution is trivial (despite your late addition of constraints) makes the problem statement suspicious. Double check it.
– Yves Daoust
Nov 15 at 10:05










2 Answers
2






active

oldest

votes

















up vote
4
down vote













Clearly,



$$a^4+a^2+7ge7$$



and equality is achieved by $a=0$ (which is not allowed).



From the given,



$$a(1+r+r^2)=a^3r^3$$



and for $rne0$,



$$a^2=frac{1+r+r^2}{r^3}$$ so that $a$ can be made a small as you want. There is no minimum, just an infimum.






share|cite|improve this answer























  • You should treat the case $a = 0$ separately, since you need $ane0$ in order to get $a^2=frac{1+r+r^2}{r^3}.$
    – Rchn
    Nov 15 at 10:04












  • @Rchn: right, rewriting. ($a>0$ is given.)
    – Yves Daoust
    Nov 15 at 10:12




















up vote
1
down vote













7



When a=b=c=0



At first I thought (and commented, and was ninja'd) that this was a spurious solution. The reason I'm now calling it an answer is that it's also the infimum of all non-degenerate answers.



Let $b=ar$ and $c=ar^2$



The condition is satisfied iff:
$a + ar + ar^2 = a^3r^3$



This gives a cubic in r: $a^2r^3-r^2-r-1=0$



This cubic is -1 at r=0, and is positive in the limit r->infinity, so it must have a positive root. Thus there exists b and c that satisfy this condition for any a. Therefore a can be chosen to be arbitrarily small.



So you can get as close to 7 as you like with "non-degenerate" answers. So either it's the answer or there is no answer.






share|cite|improve this answer





















  • See question ,a b c are positive and you are saying a=b=c=0
    – Harry Potter
    Nov 15 at 9:59










  • Due to Descarte's rule of signs has the equation $a^2r^3-r^2-r-1=0$ exactly one positive root $r.$ However, this root verifies $r>1/a$ from where $b,c>1.$
    – user376343
    Nov 15 at 10:09












  • There is no need to care about the roots of the cubic, you can work with $a=f(r)$ rather than $r=f(a)$.
    – Yves Daoust
    Nov 15 at 10:16












  • I know it @Yves. I just wanted to point out the wrong argumentation.
    – user376343
    Nov 15 at 10:26






  • 1




    @user376343: I was addressing Steven.
    – Yves Daoust
    Nov 15 at 11:10











Your Answer





StackExchange.ifUsing("editor", function () {
return StackExchange.using("mathjaxEditing", function () {
StackExchange.MarkdownEditor.creationCallbacks.add(function (editor, postfix) {
StackExchange.mathjaxEditing.prepareWmdForMathJax(editor, postfix, [["$", "$"], ["\\(","\\)"]]);
});
});
}, "mathjax-editing");

StackExchange.ready(function() {
var channelOptions = {
tags: "".split(" "),
id: "69"
};
initTagRenderer("".split(" "), "".split(" "), channelOptions);

StackExchange.using("externalEditor", function() {
// Have to fire editor after snippets, if snippets enabled
if (StackExchange.settings.snippets.snippetsEnabled) {
StackExchange.using("snippets", function() {
createEditor();
});
}
else {
createEditor();
}
});

function createEditor() {
StackExchange.prepareEditor({
heartbeatType: 'answer',
convertImagesToLinks: true,
noModals: true,
showLowRepImageUploadWarning: true,
reputationToPostImages: 10,
bindNavPrevention: true,
postfix: "",
imageUploader: {
brandingHtml: "Powered by u003ca class="icon-imgur-white" href="https://imgur.com/"u003eu003c/au003e",
contentPolicyHtml: "User contributions licensed under u003ca href="https://creativecommons.org/licenses/by-sa/3.0/"u003ecc by-sa 3.0 with attribution requiredu003c/au003e u003ca href="https://stackoverflow.com/legal/content-policy"u003e(content policy)u003c/au003e",
allowUrls: true
},
noCode: true, onDemand: true,
discardSelector: ".discard-answer"
,immediatelyShowMarkdownHelp:true
});


}
});






Harry Potter is a new contributor. Be nice, and check out our Code of Conduct.










 

draft saved


draft discarded


















StackExchange.ready(
function () {
StackExchange.openid.initPostLogin('.new-post-login', 'https%3a%2f%2fmath.stackexchange.com%2fquestions%2f2999454%2fif-a-b-c-are-in-geometric-progression-and-abc-abc-then-what-is-minimum-v%23new-answer', 'question_page');
}
);

Post as a guest















Required, but never shown

























2 Answers
2






active

oldest

votes








2 Answers
2






active

oldest

votes









active

oldest

votes






active

oldest

votes








up vote
4
down vote













Clearly,



$$a^4+a^2+7ge7$$



and equality is achieved by $a=0$ (which is not allowed).



From the given,



$$a(1+r+r^2)=a^3r^3$$



and for $rne0$,



$$a^2=frac{1+r+r^2}{r^3}$$ so that $a$ can be made a small as you want. There is no minimum, just an infimum.






share|cite|improve this answer























  • You should treat the case $a = 0$ separately, since you need $ane0$ in order to get $a^2=frac{1+r+r^2}{r^3}.$
    – Rchn
    Nov 15 at 10:04












  • @Rchn: right, rewriting. ($a>0$ is given.)
    – Yves Daoust
    Nov 15 at 10:12

















up vote
4
down vote













Clearly,



$$a^4+a^2+7ge7$$



and equality is achieved by $a=0$ (which is not allowed).



From the given,



$$a(1+r+r^2)=a^3r^3$$



and for $rne0$,



$$a^2=frac{1+r+r^2}{r^3}$$ so that $a$ can be made a small as you want. There is no minimum, just an infimum.






share|cite|improve this answer























  • You should treat the case $a = 0$ separately, since you need $ane0$ in order to get $a^2=frac{1+r+r^2}{r^3}.$
    – Rchn
    Nov 15 at 10:04












  • @Rchn: right, rewriting. ($a>0$ is given.)
    – Yves Daoust
    Nov 15 at 10:12















up vote
4
down vote










up vote
4
down vote









Clearly,



$$a^4+a^2+7ge7$$



and equality is achieved by $a=0$ (which is not allowed).



From the given,



$$a(1+r+r^2)=a^3r^3$$



and for $rne0$,



$$a^2=frac{1+r+r^2}{r^3}$$ so that $a$ can be made a small as you want. There is no minimum, just an infimum.






share|cite|improve this answer














Clearly,



$$a^4+a^2+7ge7$$



and equality is achieved by $a=0$ (which is not allowed).



From the given,



$$a(1+r+r^2)=a^3r^3$$



and for $rne0$,



$$a^2=frac{1+r+r^2}{r^3}$$ so that $a$ can be made a small as you want. There is no minimum, just an infimum.







share|cite|improve this answer














share|cite|improve this answer



share|cite|improve this answer








edited Nov 15 at 10:07

























answered Nov 15 at 9:56









Yves Daoust

121k668218




121k668218












  • You should treat the case $a = 0$ separately, since you need $ane0$ in order to get $a^2=frac{1+r+r^2}{r^3}.$
    – Rchn
    Nov 15 at 10:04












  • @Rchn: right, rewriting. ($a>0$ is given.)
    – Yves Daoust
    Nov 15 at 10:12




















  • You should treat the case $a = 0$ separately, since you need $ane0$ in order to get $a^2=frac{1+r+r^2}{r^3}.$
    – Rchn
    Nov 15 at 10:04












  • @Rchn: right, rewriting. ($a>0$ is given.)
    – Yves Daoust
    Nov 15 at 10:12


















You should treat the case $a = 0$ separately, since you need $ane0$ in order to get $a^2=frac{1+r+r^2}{r^3}.$
– Rchn
Nov 15 at 10:04






You should treat the case $a = 0$ separately, since you need $ane0$ in order to get $a^2=frac{1+r+r^2}{r^3}.$
– Rchn
Nov 15 at 10:04














@Rchn: right, rewriting. ($a>0$ is given.)
– Yves Daoust
Nov 15 at 10:12






@Rchn: right, rewriting. ($a>0$ is given.)
– Yves Daoust
Nov 15 at 10:12












up vote
1
down vote













7



When a=b=c=0



At first I thought (and commented, and was ninja'd) that this was a spurious solution. The reason I'm now calling it an answer is that it's also the infimum of all non-degenerate answers.



Let $b=ar$ and $c=ar^2$



The condition is satisfied iff:
$a + ar + ar^2 = a^3r^3$



This gives a cubic in r: $a^2r^3-r^2-r-1=0$



This cubic is -1 at r=0, and is positive in the limit r->infinity, so it must have a positive root. Thus there exists b and c that satisfy this condition for any a. Therefore a can be chosen to be arbitrarily small.



So you can get as close to 7 as you like with "non-degenerate" answers. So either it's the answer or there is no answer.






share|cite|improve this answer





















  • See question ,a b c are positive and you are saying a=b=c=0
    – Harry Potter
    Nov 15 at 9:59










  • Due to Descarte's rule of signs has the equation $a^2r^3-r^2-r-1=0$ exactly one positive root $r.$ However, this root verifies $r>1/a$ from where $b,c>1.$
    – user376343
    Nov 15 at 10:09












  • There is no need to care about the roots of the cubic, you can work with $a=f(r)$ rather than $r=f(a)$.
    – Yves Daoust
    Nov 15 at 10:16












  • I know it @Yves. I just wanted to point out the wrong argumentation.
    – user376343
    Nov 15 at 10:26






  • 1




    @user376343: I was addressing Steven.
    – Yves Daoust
    Nov 15 at 11:10















up vote
1
down vote













7



When a=b=c=0



At first I thought (and commented, and was ninja'd) that this was a spurious solution. The reason I'm now calling it an answer is that it's also the infimum of all non-degenerate answers.



Let $b=ar$ and $c=ar^2$



The condition is satisfied iff:
$a + ar + ar^2 = a^3r^3$



This gives a cubic in r: $a^2r^3-r^2-r-1=0$



This cubic is -1 at r=0, and is positive in the limit r->infinity, so it must have a positive root. Thus there exists b and c that satisfy this condition for any a. Therefore a can be chosen to be arbitrarily small.



So you can get as close to 7 as you like with "non-degenerate" answers. So either it's the answer or there is no answer.






share|cite|improve this answer





















  • See question ,a b c are positive and you are saying a=b=c=0
    – Harry Potter
    Nov 15 at 9:59










  • Due to Descarte's rule of signs has the equation $a^2r^3-r^2-r-1=0$ exactly one positive root $r.$ However, this root verifies $r>1/a$ from where $b,c>1.$
    – user376343
    Nov 15 at 10:09












  • There is no need to care about the roots of the cubic, you can work with $a=f(r)$ rather than $r=f(a)$.
    – Yves Daoust
    Nov 15 at 10:16












  • I know it @Yves. I just wanted to point out the wrong argumentation.
    – user376343
    Nov 15 at 10:26






  • 1




    @user376343: I was addressing Steven.
    – Yves Daoust
    Nov 15 at 11:10













up vote
1
down vote










up vote
1
down vote









7



When a=b=c=0



At first I thought (and commented, and was ninja'd) that this was a spurious solution. The reason I'm now calling it an answer is that it's also the infimum of all non-degenerate answers.



Let $b=ar$ and $c=ar^2$



The condition is satisfied iff:
$a + ar + ar^2 = a^3r^3$



This gives a cubic in r: $a^2r^3-r^2-r-1=0$



This cubic is -1 at r=0, and is positive in the limit r->infinity, so it must have a positive root. Thus there exists b and c that satisfy this condition for any a. Therefore a can be chosen to be arbitrarily small.



So you can get as close to 7 as you like with "non-degenerate" answers. So either it's the answer or there is no answer.






share|cite|improve this answer












7



When a=b=c=0



At first I thought (and commented, and was ninja'd) that this was a spurious solution. The reason I'm now calling it an answer is that it's also the infimum of all non-degenerate answers.



Let $b=ar$ and $c=ar^2$



The condition is satisfied iff:
$a + ar + ar^2 = a^3r^3$



This gives a cubic in r: $a^2r^3-r^2-r-1=0$



This cubic is -1 at r=0, and is positive in the limit r->infinity, so it must have a positive root. Thus there exists b and c that satisfy this condition for any a. Therefore a can be chosen to be arbitrarily small.



So you can get as close to 7 as you like with "non-degenerate" answers. So either it's the answer or there is no answer.







share|cite|improve this answer












share|cite|improve this answer



share|cite|improve this answer










answered Nov 15 at 9:55









Steven Irrgang

70126




70126












  • See question ,a b c are positive and you are saying a=b=c=0
    – Harry Potter
    Nov 15 at 9:59










  • Due to Descarte's rule of signs has the equation $a^2r^3-r^2-r-1=0$ exactly one positive root $r.$ However, this root verifies $r>1/a$ from where $b,c>1.$
    – user376343
    Nov 15 at 10:09












  • There is no need to care about the roots of the cubic, you can work with $a=f(r)$ rather than $r=f(a)$.
    – Yves Daoust
    Nov 15 at 10:16












  • I know it @Yves. I just wanted to point out the wrong argumentation.
    – user376343
    Nov 15 at 10:26






  • 1




    @user376343: I was addressing Steven.
    – Yves Daoust
    Nov 15 at 11:10


















  • See question ,a b c are positive and you are saying a=b=c=0
    – Harry Potter
    Nov 15 at 9:59










  • Due to Descarte's rule of signs has the equation $a^2r^3-r^2-r-1=0$ exactly one positive root $r.$ However, this root verifies $r>1/a$ from where $b,c>1.$
    – user376343
    Nov 15 at 10:09












  • There is no need to care about the roots of the cubic, you can work with $a=f(r)$ rather than $r=f(a)$.
    – Yves Daoust
    Nov 15 at 10:16












  • I know it @Yves. I just wanted to point out the wrong argumentation.
    – user376343
    Nov 15 at 10:26






  • 1




    @user376343: I was addressing Steven.
    – Yves Daoust
    Nov 15 at 11:10
















See question ,a b c are positive and you are saying a=b=c=0
– Harry Potter
Nov 15 at 9:59




See question ,a b c are positive and you are saying a=b=c=0
– Harry Potter
Nov 15 at 9:59












Due to Descarte's rule of signs has the equation $a^2r^3-r^2-r-1=0$ exactly one positive root $r.$ However, this root verifies $r>1/a$ from where $b,c>1.$
– user376343
Nov 15 at 10:09






Due to Descarte's rule of signs has the equation $a^2r^3-r^2-r-1=0$ exactly one positive root $r.$ However, this root verifies $r>1/a$ from where $b,c>1.$
– user376343
Nov 15 at 10:09














There is no need to care about the roots of the cubic, you can work with $a=f(r)$ rather than $r=f(a)$.
– Yves Daoust
Nov 15 at 10:16






There is no need to care about the roots of the cubic, you can work with $a=f(r)$ rather than $r=f(a)$.
– Yves Daoust
Nov 15 at 10:16














I know it @Yves. I just wanted to point out the wrong argumentation.
– user376343
Nov 15 at 10:26




I know it @Yves. I just wanted to point out the wrong argumentation.
– user376343
Nov 15 at 10:26




1




1




@user376343: I was addressing Steven.
– Yves Daoust
Nov 15 at 11:10




@user376343: I was addressing Steven.
– Yves Daoust
Nov 15 at 11:10










Harry Potter is a new contributor. Be nice, and check out our Code of Conduct.










 

draft saved


draft discarded


















Harry Potter is a new contributor. Be nice, and check out our Code of Conduct.













Harry Potter is a new contributor. Be nice, and check out our Code of Conduct.












Harry Potter is a new contributor. Be nice, and check out our Code of Conduct.















 


draft saved


draft discarded














StackExchange.ready(
function () {
StackExchange.openid.initPostLogin('.new-post-login', 'https%3a%2f%2fmath.stackexchange.com%2fquestions%2f2999454%2fif-a-b-c-are-in-geometric-progression-and-abc-abc-then-what-is-minimum-v%23new-answer', 'question_page');
}
);

Post as a guest















Required, but never shown





















































Required, but never shown














Required, but never shown












Required, but never shown







Required, but never shown

































Required, but never shown














Required, but never shown












Required, but never shown







Required, but never shown







Popular posts from this blog

QoS: MAC-Priority for clients behind a repeater

Ивакино (Тотемский район)

Can't locate Autom4te/ChannelDefs.pm in @INC (when it definitely is there)